Sunteți pe pagina 1din 25

Solutions of AILET – 2018 Held on 06-05-2018

SECTION – A
ENGLISH
Directions (Q. 1 — Q. 4): For each question, choose Solution:
the correct sentence/s. Choice ‘a’ is the right answer. In I the word
‘ingenuous’ which means innocent and
1. I. Due to the amicable nature of the new unsuspecting is not properly used. It should be
neighbours in the locality, we could come to replaced with ‘original’. In IV the words
an amiable settlement of the dispute that had abjure(solemnly renounce (a belief, cause, or
taken Place. claim) and adjure (urge or request (someone)
II. Edmund Hillary and Tenzing Norgay gave solemnly or earnestly to do something) should
ascent that they Will assent the Mt. Everest change places. Choice (a)
and conquer it together.
III. His immanent literary qualities made him an 3. (a) Ketan has to be more discreet or he'll literally
eminent writer and hence fame became drive his wife up the wall.
imminent for him. (b) Ketan has to be more discrete or he'll literally
IV. Kamal is very obedient, agreeable and drive his wife up the wall.
gracious: a complaisant child. His mother (c) Ketan has to be more discrete or he'll drive his
feels complacent of her parentage. wife up the wall.
(d) Ketan has to be more discreet or he'll drive his
(a) I, IV (b) III, IV (c) II, IV (d) I, III wife up the wall.

Solution: Solution:
Choice ‘b’ is the right answer. Sentences III and Sentences b and c are incorrect. The word
IV are correct. In I the phrase ‘due to’ is usually ‘discrete’ means ‘individually separate and
not used at the beginning of a sentence. It should distinct’. That meaning is not suitable in this
be replaced with ‘owing to’. “Due to" means context. Between a and d the letter is correct.
"caused by", however, "Owing to" means Choice (d)
"because of”. In II the construction ‘gave ascent’ is 4. (a) Hema prophesized that if Ratna kept dancing
incorrect. Ascent means ‘a climb or walk to the that way, she'd get her just desserts.
summit of a mountain or hill’ and it is a noun and (b) Hema prophesized that if Ratna kept dancing
not a verb. that way, she'd get her just deserts.
Choice (b) (c) Hema prophesied that if Ratna kept dancing
that way, she'd get her just deserts.
2. I. The Music Director ingeniously told the media (d) Hema prophesied that if Ratna kept dancing
that the song was his ingenuous work that way, she'd get her just desserts.
II. lf you are mendacious in life most of the times,
you Will have to be mendicant later to win their Solution:
trust back again. Sentences ‘a, b’ and ‘d’ are incorrect. The word
III. Being venal is not a venial crime under the “prophesize”, according to most dictionaries, it isn't
court of law. a word at all.; 'b' and c contain the words ‘deserts’
IV. The leader abjured his followers to adjure which means what a person deserves with regard to
violence as a means to demand their rights. reward or (more usually) punishment and ‘desserts’
means the sweet course eaten at the end of a meal.
(a) II, III (b) I, IV (c) II, IV (d) I, III ‘c’ is the correct answer. Choice (c)

Directions (Q. 5 — Q. 6): Following are the questions based on the same words used as different parts of speech.
Choose the correct matches.

5. Well
1. Noun a. Well begun is half done.
2. Adjective b. Let well alone.
3. Adverb c. Well, who would have thought it?
4. Interjection d. I hope you are now well.

(a) I-a, 2-b, 3-c, 4-d (b) I-b, 2-d, 3-a, 4-c (c) I-c, 2-d, 3-a, 4-b (d) 4-a, 3-b, 2-c, I-b

Solution:
Choice ‘b’ is the right answer. In 'a’ well is an adverb qualifying the verb ‘begin’; In 'b' it is a noun; In 'c' well is
an interjection, in 'd' it is an adjective. Choice (b)
Triumphant Institute of Management Education Pvt. Ltd. (T.I.M.E.) HO: 95B, 2nd Floor, Siddamsetty Complex, Secunderabad – 500 003.
Tel : 040–27898195 Fax : 040–27847334 email : info@time4education.com website : www.time4education.com AILET2018/1
6. What

1. Interjection Adjective a. Give me what you can. What happened then, I do not know.

2. Interjection b. What does he want?

3. Interjection Pronoun c. What! You don't mean to say so?


4. Relative Pronoun d. What evidence have you got?

(a) 4-b, I-c, 2-d, 3-a (b) 3-c, 2-b, I-a, 4-d (c) I-c, 2-b, 3-d, 4-a (d) I-d, 2-c, 3-b, 4-a

Solution:
Choice 'd' is the right answer. In 'a' what is a relative pronoun in the first part of the sentence. In 'b' ‘what’ is an
interjection pronoun, in 'c' ‘what’ is an interjection and in d ‘what’ is an interjection adjective.
Choice (d)

Directions (Q. 7 — Q. 12): Name the various Figures represent the whole or vice versa. Here
of Speech in the following. ‘brandished steel’ may represent the sword
Macbeth holds or it may be the steel like body of
7. He was a learned man among lords, and a lord Macbeth or it may be the steel like will power of
among learned men. Macbeth. Choice (d)
(a) Epigram (b) Metonymy
(c) Oxymoron (d) Antithesis 10. We had nothing to do, and we did it very well.
(a) Antithesis (b) Paradox
Solution: (c) Anticlimax (d) Litotes
Choice a is the right answer. Epigram is a
rhetorical device that is a memorable, brief, Solution:
interesting, and surprising satirical statement. Choice 'b' is the right answer. A paradox is a figure
Another example is: “Mankind must put an end to of speech in which a statement appears to
war, or war will put an end to mankind.” contradict itself. Choice (b)
Choice (a)
11. And thou, Dalhousie, the great god of war
8. Sceptre and crown Lieutenant-Colonel to the Earl of Mar.
Must tumble down, (a) Apostrophe (b) Epigram
And in dust be equal made (c) Anticlimax (d) Paradox
With the poor crooked scythe and spade.
Solution:
(a) Irony Choice 'c' is the right answer. Anticlimax is a figure
(b) Apostrophe of speech in which statements gradually descend
(c) Metonymy in order of importance. Here Dalhousie is
(d) Synecdoche considered to be ‘the great god of war’ and later as
a Lieutenant Colonel to the Earl of Mar. Thus, the
Solution: titles given descend in order of importance.
Choice 'c' is the right answer. Metonymy is a figure Another example is: She is a great writer, a mother
of speech in which one word or phrase is and a good humorist.
substituted for another with which it is closely Choice (c)
associated; also, the rhetorical strategy of
describing something indirectly by referring to 12. The Puritan had been rescued by no common
things around it. Here ‘sceptre (an ornamented deliverer from the gasp of no common foe.
staff carried by rulers on ceremonial occasions as (a) Hyperbole
a symbol of sovereignty.) and crown represent a (b) Epigram
king. Similarly scythe and spade represent a (c) Metaphor
common man or a worker. (d) Litotes
Choice (c)
Solution:
9. Brave Macbeth, with his brandished steel, carved Choice 'd' is the right answer. Litotes is a figure of
out his passage. speech that employs an understatement by using
(a) Metaphor (b) Litotes double negatives or, in other words, a positive
(c) Climax (d) Synecdoche statement expressed by negating its opposite
expressions. The double negatives are ‘no
Solution: common deliverer’ and ‘ no common foe’ in this
Choice d is the right answer. Synecdoche is a example.
figure of speech in which a part is made to Choice (d)
Triumphant Institute of Management Education Pvt. Ltd. (T.I.M.E.) HO: 95B, 2nd Floor, Siddamsetty Complex, Secunderabad – 500 003.
Tel : 040–27898195 Fax : 040–27847334 email : info@time4education.com website : www.time4education.com AILET2018/2
Directions (Q. 13 — Q. 19): Choose the sentence (s) Solution:
which is/are punctuated correctly. Choice 'b' is the right answer. In II there is no
comma between the two elements of the
13. I. Our daughter will be three years old next interjection.
week. Choice (b)
II. Our son will be two-years-old next week.
III. Our two-year-old is starting to talk. 17. I. As Caesar loved me, I wept for him; as he was
IV. Our two-year-old is starting to talk. fortunate, I rejoice at it; as he was valiant, I
honour him; but as. he was ambitious, I slew
(a) I, III (b) I & IV (c) II, III (d) III him.
II. Danish, my friend from Chandigarh, Punjab,
Solution: will join us.
Choice 'a' is the right answer. In II there is no III. Danish, my friend, from Chandigarh, Punjab
necessity of a hyphen between ‘two years’. It is will join us.
necessary to keep a hyphen between ‘two' and IV. As Caesar loved me, I wept for him; as he was
'year’. Choice (a) fortunate, I rejoice at it; as he was valiant, I
honour him: but as he was ambitious I slew
14. I. Jan asked; "What did Joe mean when he said, him.
'l will see you later.' "
II. Jan asked, "What did Joe mean when he said, (a) II (b) II, IV (c) I, III (d) I, II
'l will see you later?' "
III. Jan asked, "What did Joe mean when he said, Solution:
'l will see you later'?" Choice 'd' is the right answer. In III the comma
IV. Jan asked, "What did Joe mean when he said, after ‘my friend’ is incorrect. In IV there should be
'l will see you later' "? a comma after ‘but’ or after the clause ‘as he was
ambitious’.
(a) II (b) I (c) IV (d) III Choice (d)

Solution: 18. I. A liberally sprinkled dose of humour was very


Choice 'd' is the right answer. The sentence within much appreciated.
the inverted commas is an interrogative question. II. Right now I want two things: peace and quiet.
It should end with a question mark. The question III. Our liberal-minded clergyman managed to
mark should be within the inverted commas. unite the entire congregation.
Choice (d) IV. Right now I want two things; peace and quiet.
15. I. You are my friend; however, I cannot afford to (a) II, III (b) I, IV (c) I, II, III (d) III, IV
lend you any more money.
II. Truly, a popular error has as many lives as a Solution:
cat: it comes walking in, long after you have
Choice 'c' is the right answer. In 'IV' the semi-colon
imagined it effectually strangled.
should be replaced with a colon. A semicolon
III. There is only one cure for the evils which
can be used as a super-comma, to separate items
newly acquired freedom produces, and that
in a long list. The colon is most commonly used to
cure is freedom.
introduce and can be used to introduce anything:
IV. There is a slavery that no legislation can
words, phrases, lists, names or quotations.
abolish; the slavery of caste.
Choice (c)
(a) I, II, III (b) II, IV
(c) l, II, III, IV (d) II, III 19. I. It may not be the correct part, but I bet that it
works.
Solution: II. It may not be the correct part but: I bet that it
Choice 'a' is the right answer. In IV the semicolon works.
is not appropriate. A semi colon serves to III. O father! I hear the sound of guns.
eliminate the pause between the two independent IV. O Hamlet, speak no more!
clauses and is therefore used instead of a
conjunction. (a) I, III (b) I, III, IV (c) II, III (d) I, IV
Choice (a)
Solution:
16. I. Yes, Jagrit, you were right about that answer. Choice 'b' is the right answer. The colon in II is
II. Yes Jagrit, you were right about that answer. incorrect. There should be a semi-colon because
III. I saw our town's ex-Mayor Prakash Kumar in two clauses are separated here. A semi-colon
the mall. serves to eliminate the pause between the two
IV. I saw our town's ex-mayor in the market. independent clauses and is therefore used instead
of a conjunction.
(a) I, III (b) I, III, IV (c) I, IV (d) II, III Choice (b)

Triumphant Institute of Management Education Pvt. Ltd. (T.I.M.E.) HO: 95B, 2nd Floor, Siddamsetty Complex, Secunderabad – 500 003.
Tel : 040–27898195 Fax : 040–27847334 email : info@time4education.com website : www.time4education.com AILET2018/3
Directions (Q. 20 — Q. 26): For each question, (d) This gorgeous grand sitar is tuned to perfection.
identify the correct subjects and verbs. The subjects
are in bold and the verbs are underlined. Solution:
Choice b is the right answer. Choice (b)
20. (a) Every environmental regulation has been
undermined by that industry. Directions (Q. 27 — Q. 30): Find out the sentence/s
(b) Every environmental regulation has been which is/are written correctly.
undermined by that industry.
I. Some of the desserts was left by the end of the
(c) Every environmental regulation has been
birthday party.
undermined by that industry.
(d) Every environmental regulation has been II. Your brilliant excuses almost makes up for your
tardiness.
undermined by that industry.
III. Neither Jackson nor Jenna have played hooky.
Solution: IV. Either of us is capable of winning.
Choice c is the right answer. Choice (c)
27. (a) I, II, III (b) II, III (c) II (d) IV
21. (a) My gift for singing and dancing
simultaneously did not fail to attract attention. Solution:
(b) My gift for singing and dancing simultaneously Choice 'd' is the right answer. In 1 the subject is
did not fail to attract attention. plural and so it needs a plural verb ‘were’ and ‘was’
(c) My gift for singing and dancing is incorrect. In II the plural subject ‘excuses’ takes
simultaneously did not fail to attract attention. a plural verb, but the verb ‘makes’ is singular. It
(d) My gift for singing and dancing should be corrected as ‘make’. The usage ’Neither
simultaneously did not fail to attract attention. – nor’ in III takes a singular verb. The correction is
‘has’ in the place of ‘have’. Choice (d)
Solution:
Choice d is the right answer. Choice (d) I. Here's the paper clips you requested.
II. A limousine and driver are in the driveway.
22. (a) Every attempt to flatter her failed miserably. III. One-fourth of the voters tend not to cast its ballots
(b) Every attempt to flatter her failed miserably. in national elections.
(c) Every attempt to flatter her failed miserably. IV. Either of the classic cars are for sale.
(d) Every attempt to flatter her failed miserably.
28. (a) I, II (b) I, II, III (c) II (d) II, IV
Solution:
Choice c is the right answer. Choice (c) Solution:
Choice 'c' is the right answer. In I the first verb
23. (a) If all is lost, why am I still playing cricket? should be in plural because its real subject is
(b) If all is lost, why am I still playing cricket? ‘paper clips’ a plural noun. The correction is ‘Here
(c) If all is lost, why am I still playing cricket? are the ….’ In III the subject, one fourth of the
(d) If all is lost, why am I still playing cricket? voters is not a non-living thing, but human beings.
The pronoun to be used to refer this subject should
Solution: be ‘their’ and not ‘its’. When the two nouns are
Choice a is the right answer. Choice (a) connected with ‘Either …. or’, the verb should be
in singular. The correction is, replace ‘are’ with ‘is’.
24. (a) "Buses" has only one s in the middle of it. Choice (c)
(b) "Buses" has only one s in the middle of it.
(c) "Buses" has only one s in the middle of it. I. Ten dollars are too much to pay for a pen.
(d) "Buses" has only one s in the middle of it. II. The water polo team have won the state
championship for the second time.
Solution: III. That skier is one of those who competes nationally.
Choice d is the right answer. Choice (d) IV. I cannot be calm on roller coasters.
25. (a) Have you memorized all the chemical 29. (a) I, II, III (b) II, III, IV
symbols on the chart? (c) II, IV (d) none of these
(b) Have you memorized all the chemical
symbols on the chart? Solution:
(c) Have you memorized all the chemical Choice 'd' is the right answer. All the sentences are
symbols on the chart? grammatically incorrect. In I ‘ten dollars’ is a single
(d) Have you memorized all the chemical symbols entity and takes singular verb. ‘Is must be used in
on the chart? place of ‘are’. In II, ‘the water polo team’ is singular
and so takes singular verb. The verb should be
Solution: ‘has won’ and not ‘have won’. In III the subject is
Choice a is the right answer. Choice (a) ‘those’ and not ‘one’. So, the verb should be in
plural. ‘Competes’ should be replaced with
26. (a) This gorgeous grand sitar is tuned to perfection. ‘compete’. In IV, using ‘cannot’ as two words is
(b) This gorgeous grand sitar is tuned to perfection. incorrect. It should be used as a single word
(c) This gorgeous grand sitar is tuned to perfection. ‘cannot’. Choice (d)
Triumphant Institute of Management Education Pvt. Ltd. (T.I.M.E.) HO: 95B, 2nd Floor, Siddamsetty Complex, Secunderabad – 500 003.
Tel : 040–27898195 Fax : 040–27847334 email : info@time4education.com website : www.time4education.com AILET2018/4
I. The sexy, slim, Indian superstar. 33. (To) Go cold turkey
II. The Indian, slim, sexy superstar. (a) speaking truth
III. My yellow, old, Ferrari sports car. (b) getting down to business
IV. My old, yellow, Ferrari sports car. (c) gradually cutting down
(d) withdraw abruptly and completely
30. (a) I, IV (b) I, III (c) II, IV (d) II, III
Solution:
Solution: Choice 'd' is the right answer. To “go cold turkey”
Choice 'a' is the right answer. The sequence of is to withdraw suddenly and completely from an
adjectives is: Opinion, size, shape, age, colour, addictive substance or some other form of
nationality/origin and material. This sequence is dependency. Choice (d)
not followed in II and III. Choice (a)
34. Break a leg
Directions (Q. 31 — Q. 35): Choose the alternative (a) good luck
which best expresses the meaning of the idiom/ phrase (b) take a break
in the question. (c) bad luck
(d) trying hard to succeed
31. Find your feet
(a) desire to escape from a situation Solution:
(b) adapting and attuning to something Choice 'a' is the right answer. The idiom, break a
(c) finding one's own space leg is used to encourage and typically said to one
(d) ready to fight a war who is about to perform before an audience,
especially a theater actor. It is thought to be used
Solution: due to the superstition that wishing one "good
Choice 'b' is the right answer. The idiom ‘find your luck" will result in the opposite. Choice (a)
feet’ means to become familiar with and confident
in a new situation. Choice (b) 35. White-knuckled handshake
(a) sign of stressful situation sign of peace
32. (To be) On the ball (b) friendly handshake
(a) another thing coming (c) sign of peace
(b) having fun (d) holding hands very lightly
(c) being alert or well prepared
(d) raise the roof Solution:
Choice 'a' is the right answer. ‘White knuckle’
Solution: implies something marked by, causing, or
Choice 'c' is the right answer. The idiom ‘to be on experiencing tense nervousness. A white
the ball’ means to be knowledgeable and knuckled handshake is a handshake from which
competent. alert, in command of senses, attentive. tension and nervousness can be observed. Hence
understands the situation well, quick to such a handshake indicates a stressful situation.
understand and react to a situation. Choice (a)
Choice (c)

SECTION – B
GENERAL KNOWLEDGE
36. Voter Verifiable Paper Audit Trail (VVPAT) is an Foreign Language category at the 90th Academy
independent verification system for EVM designed Awards (Oscars).
to (a) Padmavat (b) Dangal
I. allow voters to verify that their votes are cast (c) Bahubali (d) Newton
correctly.
II. detect possible election fraud or malfunction. Answer key:
III. Provide a means to audit the stored electronic Choice (d)
results.
IV. Voter can see printout and take it out too. 38. Who is awarded the Person of the Year for 2017
by Times magazine?
(a) I and II (b) I, III and IV (a) The Silence Breakers
(c) I, II and III (d) All of the above (b) Crown Prince Mohammed bin Salman
(c) The Dreamers
Answer key: (d) Colin Kaepernick
Choice (c)
Answer key:
37. The Film Federation of India (FFI) has selected the Choice (a)
Hindi film _______ to represent India in Best
Triumphant Institute of Management Education Pvt. Ltd. (T.I.M.E.) HO: 95B, 2nd Floor, Siddamsetty Complex, Secunderabad – 500 003.
Tel : 040–27898195 Fax : 040–27847334 email : info@time4education.com website : www.time4education.com AILET2018/5
39. How many banks have merged with SBI with effect Answer key:
from October 2017? Choice (c)
(a) 4 (b) 5 (c) 6 (d) 7
46. Which of these digital payment mechanisms does
Answer key: not require an internet connection?
Choice (c) (a) USSD (b) UPI
(c) e-wallets (d) IMPS 24X7
40. According to a recent report by NASA, the El Nino
led to excessive carbon dioxide releases in which Answer key:
of the following ways? Choice (a)
I. Hot weather and drought caused extensive
wildfires in south-east Asia. 47. ______ is a popular statistical simulation named
II. Drought in the Amazon rainforest stunted after a city.
plant growth, reducing the amount of carbon (a) Bristol (b) Vienna
they absorb while growing. (c) Berlin (d) Monte Carlo
III. Warmer weather and near normal rainfall in
Answer key:
Africa caused forests to exhale more Co2.
Choice (d)
(a) I only (b) II only 48. BookMyShow and Paytm are examples of ______
(c) II and III (d) I, II and III and ______ wallets, respectively.
(a) Open, Closed
Answer key: (b) Closed, semi-closed
Choice (d) (c) Semi-open, Semi-closed
(d) Semi-closed, closed
41. Scientists from which of the following countries
have developed the world's first artificial Answer key:
intelligence politician named SAM? Choice (b)
(a) New Zealand (b) Japan
(c) China (d) Korea 49. Which of the following Indian companies had
become one of the top three most-valuable brands
Answer key: in the information technology (IT) service sector of
Choice (a) the world?
(a) Wipro
42. Which State became the first State in India to (b) Infosys
operate electric bus services? (c) HCL Technologies
(a) Assam (b) Himachal Pradesh (d) Tata Consultancy services
(c) Karnataka (d) Maharashtra
Answer key:
Answer key: Choice (d)
Choice (b)
50. ICC Under-19 World Cup Trophy 2018 was won
43. ______ officially launched its own oil-backed by India defeating Australia. What is special about
cryptocurrency called Petro. this tournament ?
(a) China (b) Senegal (a) Shubman Gill of India was declared the 'Man
(c) Venezuela (d) Singapore of the Tournament'.
(b) Team Afghanistan defeated three high profile
Answer key: teams namely Pakistan, Sri Lanka and New
Choice (c) Zealand.
(c) India now holds the most wins record in
Under-19 World Cup.
44. ______ is the first Indian woman pilot of Indian Air
(d) All of the above
Force who created history by completing a solo
flight on a MiG-21 Bison fighter aircraft.
Answer key:
(a) Bhawana Kanth (b) Mohana Singh
Choice (d)
(c) Anny Divya (d) Avani Chaturvedi
51. When ghee is kept in open air/sunlight for 30
Answer key: minutes, the loss of Vitamin 'A' is
Choice (d) (a) 100%
(b) 75%
45. ______ is first city in India to get World Heritage (c) 34%
City status by United Nations Educational, (d) None of the above
Scientific and Cultural Organization (UNESCO).
(a) Delhi (b) Mumbai Answer key:
(c) Ahmedabad (d) Jaipur Choice (a)

Triumphant Institute of Management Education Pvt. Ltd. (T.I.M.E.) HO: 95B, 2 Floor, Siddamsetty Complex, Secunderabad – 500 003.
nd

Tel : 040–27898195 Fax : 040–27847334 email : info@time4education.com website : www.time4education.com AILET2018/6


52. Which one of the following has been appointed WTO parlance as _______ support as a
first female independent Director of International prerequisite for consideration of other reforms in
Cricket Council in 2018? domestic support negotiations.
(a) Naina Lal Kidwai (a) Amber Box (b) Green Box
(b) Indra Nooyi (c) Blue Box (d) Orange Box
(c) Nita Ambani
(d) Chitra Ramkrishna Answer key:
Choice (a)
Answer key:
Choice (b) 59. Which of the following statements is correct as per
the announcement of Union Budget 2018-19?
53. Light waves can be polarised because they (a) 24 new Medical Colleges will be established
(a) have high frequency as green field projects.
(b) have short wavelength (b) 24 new Medical Colleges to be established by
(c) are compressive upgrading leading district hospitals.
(d) are transverse (c) 24 new Medical Colleges to be established in
tribal areas.
Answer key: (d) 24 new Medical Colleges to be established for
Choice (d) girl students.

54. As per NIT! Aayog's Health Index, 2018, which of Answer key:
the following States is the healthiest State in India? Choice (b)
(a) Punjab (b) Haryana
(c) Kerala (d) Tamil Nadu 60. Which has been termed as the 'Green Gold' by the
Finance Minister?
Answer key: (a) Shell Gas
Choice (c) (b) CNG
(c) Bamboo
55. Which one of the following has been described as (d) Medicinal and Aromatic plants
a 'Frozen Moment in History'?
(a) Fatehpur Sikri Answer key:
(b) Jallianwala Bagh Choice (c)
(c) Gol Gumbas Bijapur
(d) Kutub Minar 61. Which of the following statements is not true about
UNAIDS' target 90 : 90 : 90 ?
Answer key: (a) 90% control of HIV infection by the year 2020.
Choice (a) (b) 90% of all people living with HIV will know their
HIV status.
56. In a coastal city, a high tide occurs every (c) 90% of all people with diagnosed HIV infection
(a) 24 hours will receive sustained anti-retroviral therapy.
(b) 12 hours and 25 minutes (d) 90% of all people receiving anti-retroviral
(c) 12 hours therapy will have viral suppression.
(d) 24 hours and 25 minutes
Answer key:
Answer key: Choice (a)
Choice (b)
62. According to John Templeton — Bull markets are
57. Which of the followings are the exclusive powers born on pessimism, grow on scepticism, mature on
of Rajya Sabha under the Constitution of India? optimism and die on ______.
l. Creation of new All India Services. (a) Excitement (b) Euphoria
ll. Enable Parliament to enact law on a subject in (c) Ebullition (d) Ravishment
State List.
III. Enforcing proclamation of emergency when Answer key:
Lok Sabha is dissolved. Choice (b)

(a) I, II (b) II, Ill (c) I, III (d) I, II, III 63. If RBI lowers the Cash Reserve Ratio (CRR), then
I. supply of money in the economy may increase.
Answer key: II. banks will increase the interest rates.
Choice (d) III. exports will become cheaper.

58. India and China jointly submitted a proposal to the (a) I (b) I, II (c) I, III (d) I, II, III
World Trade Organisation (WTO) calling for the
elimination - by developed countries - of the most Answer key:
trade-distorting form of farm subsidies, known in Choice (a)
Triumphant Institute of Management Education Pvt. Ltd. (T.I.M.E.) HO: 95B, 2 Floor, Siddamsetty Complex, Secunderabad – 500 003.
nd

Tel : 040–27898195 Fax : 040–27847334 email : info@time4education.com website : www.time4education.com AILET2018/7


64. GST has a ______ tier structure for taxation of (c) valid
goods and services. (d) valid but subject to the approval of the
(a) Two Parliament
(b) Three
(c) Four Answer key:
(d) Five Choice (c)

Answer key: 68. In 2018, scientists have discovered a new organ


Choice (c) which could also be the biggest organ in human
body. What is the name of the organ?
65. Which of the following statements is correct about (a) Interstitium
FDI in India? (b) Mesentery
(a) 100% FDI is allowed in single brand retail via (c) Langerhans
automatic route (d) Myeloid
(b) 100% FDI is admissible in construction sector
via automatic route Answer key:
(c) Foreign airlines are allowed to invest upto Choice (a)
49% in Air India through approval route
(d) All of the above 69. World's longest sandstone cave has been
discovered in which of the following States?
Answer key: (a) Assam
Choice (d) (b) Meghalaya
(c) Jammu & Kashmir
66. 'Diopter' is the unit of measurement of _______. (d) Nagaland
(a) Heat
(b) Sound Answer key:
(c) Energy Choice (b)
(d) power of a lens
70. Which is the first Indian city to have its own logo
Answer key: for promoting tourism?
Choice (d) (a) Goa
(b) Agra
67. If the election of the President of India is declared (c) Jaipur
void by the Supreme Court, the acts performed by (d) Bengaluru
the President incumbent before the date of such
decision of court are Answer key:
(a) valid but subject to judicial review Choice (d)
(b) invalid

SECTION – C
LEGAL APTITUDE
Directions: Apply the legal principles to the facts given about a recurrence of thrombosis. Anil and Reena
below and select the most appropriate answer. are claiming compensation for their injuries which
were caused due to the negligence of the shop
owners. The shop owners are denying liability on
71. Legal Principle:
the grounds of volenti non-fit injuria. The defence
1. A person is liable for his negligence when he of volenti non-fit injuria (a) is available in respect
owed a duty of care to others and commits a of husband
breach of that duty causing injury thereby.
(b) is available in respect of wife
2. Volenti non-fit injuria is defence to negligence.
(c) is available in respect of both husband and
wife
Factual Situation: Anil and his wife, Reena, were (d) is not available in respect of both husband and
in a shop as customers, where a skylight in the wife
roof of the shop was broken, owing to the
negligence of the contractors engaged in repairing
the roof, and a portion of the glass fell and struck Solution:
Anil causing him a severe shock. Reena, who was As per the above stated principle Volenti Non-fit
standing close to him, was not touched by the Injuria, it can be used as a defence where the risk
falling glass, but, reasonably believing her is foreseen, but here it is not applicable, as it was
husband to be in danger, she instinctively clutched not foreseen, and they never consented to take the
his arm, and tried to pull him from the spot. In doing risk of falling of skylight in the shop.
this, she strained her leg in such a way as to bring Choice (d)
Triumphant Institute of Management Education Pvt. Ltd. (T.I.M.E.) HO: 95B, 2nd Floor, Siddamsetty Complex, Secunderabad – 500 003.
Tel : 040–27898195 Fax : 040–27847334 email : info@time4education.com website : www.time4education.com AILET2018/8
Directions (Q. 72 — Q. 73): Apply the legal principles to contamination of a water borehole just 3
to the facts given below and select the most miles away.
appropriate answer.
Solution:
Legal Principles: As per the Principle no. 3, it is clear that a person
1. Private nuisance is a continuous, unlawful and is liable if he could reasonably foresee the damage
indirect interference with the use or enjoyment of caused to the party, but so far, as here in the
land, or of some right over or in connection with it. present case, it is has not been foreseen for
2. The person who for his own purposes brings on tannery, that the spillage may eventually lead to
his lands and collects and keeps there anything contamination of water. Choice (b)
likely to do mischief if it escapes, must keep it at
his peril, and, if he does not do so, is prima facie 73. Factual Situation: M G Ltd. was constructing
answerable for all the damage which is the natural Crystal Heights, a posh state-of-the-art tower for
consequence of its escape. commercial and residential purposes, in
3. A person is liable if he can reasonably foresee that Gurugram. During construction, hundreds of
his acts would likely to injure his neighbour. claimants alleged that, in addition to dust and
4. The foreseeability of the type of damage is a pre- noise caused by the erection of the building, their
requisite of liability in actions of nuisance. television signals had been interrupted by the
tower. The claimants, some of whom were
72. Factual Situation: During 2011, a European absolute owners, and many others who were
Directive was issued requiring nations of the renting, sued in both negligence and in nuisance
European Community to establish standards on for the harm done to their amenity by the loss of
the presence of Perchloroethene (PCE) in water, their television signals. Whether the respondent's
which the Kingsland did in 2013. action in causing the appellant's television signals
to be interrupted with the construction of their
Alfa Water Co. purchased a borehole in 2007 to tower could constitute a private nuisance?
extract water to supply to the public in Kingsland. (a) The interference with the television signal
In 2014, it tested the water to ensure that it met caused by the construction of the tower could
minimum standards for human consumption and not amount to a private nuisance at law.
discovered that it was contaminated with an Effective town planning can sort this matter,
organochlorine solvent (PCE). On investigation, it instead.
emerged that the solvent seeped into the soil (b) Yes, the large tower had interrupted their
through the building floor of the Light & Soft television reception, and caused private
Leather Tannery, about 3 miles from the borehole nuisance — for loss of enjoyment — and
that eventually contaminated the Alfa's borehole. remuneration for their wasted television
license fee, from the time their signal had been
Since the tannery opened in 1910, until 2007, the impaired.
solvent it used had been delivered in 40-gallon (c) No, it cannot constitute private nuisance, but
drums which were transported by fork lift truck and the claimants can claim damages for loss of
then tipped into a sump. Since 2007, solvents had television signals.
been delivered in bulk and stored in tanks. It was (d) Yes, the respondent's conduct was
then piped to the tanning machinery. There was no unreasonable because the act of building the
evidence of any spills from the tanks or pipes, and tower caused impairment of enjoyment of the
it was concluded that the water had been land.
contaminated by frequent spills under the earlier
system. Alfa Water brought a claim against the Solution:
Tannery on the grounds of nuisance, Whether the As per the above stated principle no. 1, the
Tannery owners are liable? construction of a tower is not amounting to private
(a) Yes, the escape of the solvent which nuisance as there is a clear condition in the
contaminated the water is sufficient for making principle that there has to be a continuation in the
them liable. act, but here the act is temporary and could be
(b) No, the damage is too remote as it was not resolved after construction. Choice (a)
possible for the Tannery owners to reasonably
foresee a spillage which would eventually lead 74. Legal Principle:
to contamination of a water borehole so far 1. Any intentional false communication, either
away. written or spoken, that harms a person's
(c) No, because Alfa Water Co. should have been reputation; decreases the respect, regard, or
careful in using good purifying mechanisms to confidence in which a person is held; or
ensure that the water is fit for human induces disparaging, hostile, or disagreeable
consumption. They cannot shift the blame on opinions or feelings against a person.
the Tannery owners. 2. The statement must tend to lower the claimant
(d) Yes, the damage is not remote as it was in the estimation of right-thinking members of
possible for the Tannery owners to reasonably society.
foresee a spillage which would eventually lead 3. A mere vulgar abuse is not defamation.

Triumphant Institute of Management Education Pvt. Ltd. (T.I.M.E.) HO: 95B, 2nd Floor, Siddamsetty Complex, Secunderabad – 500 003.
Tel : 040–27898195 Fax : 040–27847334 email : info@time4education.com website : www.time4education.com AILET2018/9
4. Sometimes a statement may not be point of contention was decreasing the respect,
defamatory on the face of it but contain an inducing disparage to her, and as per principle no.
innuendo, which has a defamatory meaning. 4, when she compared Mrs. Anshika Chauhan
5. Defamation encompasses both written with Asha, so it doesn’t look like defamation, but it
statements, known as libel, and spoken is innuendo. As per principle no. 2 these tweets
statements, called slander. are sufficient enough to lower the thinking of a
prudent man towards, both Asha, and Anshika.
Factual Situation: In May 2017, a memorial Choice (a)
commemorating the women of World War Il was
vandalized during an anti-government 75. Legal Principle:
demonstration following the General Election. An 1. An assault is an act which intentionally causes
offensive political slogan was spray painted across another person to apprehend the infliction of
the plinth of the memorial. This act caused public immediate, unlawful force on a person.
outrage and widespread condemnation. 2. A battery consists of an intentional application
of force to another person without any lawful
On Twitter, a political writer, Asha Mehta said that justification.
she did not have a problem with the vandalism of
Factual Situation: Jagan was in his car when he
the memorial building. Chandna reacted to this
was approached by a police officer who told him to
negatively, suggesting that Asha should be sent to
move the vehicle. Jagan did so, reversed his car
join Terrorist Organization. Asha's comments and
and rolled it on to the foot of the police officer. The
Chandna's reactions both received national media
officer forcefully told him to move the car off his
coverage.
foot at which point Jagan swore at him and refused
to move his vehicle and turned the engine off.
A few days later, Chandna published a tweet
Jagan was convicted for assaulting a police officer
asking the question "Scrawled on any war
in the execution of his duty. Is he liable for battery
memorials recently?" to Anshika Chauhan,
or assault?
another political activist. Anshika Chauhan
(a) He is not liable because there cannot be an
responded stating that they had never vandalised
assault in omitting to act and that driving on to
any memorial building, and moreover had family
the officer's foot was accidental, meaning that
members serving in the armed forces. Chandna
he was lacking mens rea when the act causing
followed with a second tweet, in which she asked
damage had occurred.
if someone could explain the difference between
(b) He is not liable as the act neither amount to an
Mehta (an "irritant") and Anshika Chauhan (whom
attempt nor a threat to commit a battery that
she described as "social anthrax").
amounts to an actionable tort of assault.
(c) Jagan's crime was not the refusal to move the
Anshika Chauhan asked for a retraction via Twitter
car but that of having driven on to the foot of
and was promptly blocked by Chandna. Anshika
the officer and decided not to cease the act,
Chauhan asked Chandna to make a public
he had established a continual act of battery.
apology and claimed compensation for libel
(d) He is neither liable for assault nor battery as
alleging that the First Tweet suggested that she
he accidently drove his car on the police
had either vandalised a war memorial, which was
officer's foot.
a criminal act; and the Second Tweet suggested
that she approved or condoned that vandalisation.
Solution:
What is the meaning of the Tweets and whether
As per above stated principle, the act of Jagan
those meanings had defamatory tendency?
amounts to battery as whatever he has done, fulfils
(a) Both the tweets were defamatory to Anshika
all the conditions of principle 2, as not removing
Chauhan as the hypothetical ordinary reader
the car shows his intention to harm the police
can be expected to understand defamatory
officer, and he was not justified by the law to do
tendency of the tweet in the context of the
so. Choice (c)
situation.
(b) Second tweet was not defamatory as it was Directions (Q. 76 — Q. 78): Apply the legal principles
not referring directly to Anshika Chauhan. So, to the facts given below and select the most
she cannot claim compensation. appropriate answer.
(c) Natural and ordinary meaning of the tweets
are not defamatory. So, she cannot claim Legal Principles:
compensation. 1. Consideration is something that moves from the
(d) First tweet was not defamatory because the promise to the promisor, at the implied or express
natural and ordinary meaning of the statement request of the latter, in return for his promise. The
which is conveyed to a hypothetical ordinary item that moves can be a right, interest, profit, loss,
reader is not defamatory. responsibility given or suffered, forbearance or a
benefit which is of some value in the eyes of law.
Solution: 2. An offer may be revoked at any time before the
As per the above stated principle no. 1, it is clear communication of its acceptance is complete as
that Chandana defamed Mrs. Asha Mehta, as her against the proposer, but not afterwards.
Triumphant Institute of Management Education Pvt. Ltd. (T.I.M.E.) HO: 95B, 2nd Floor, Siddamsetty Complex, Secunderabad – 500 003.
Tel : 040–27898195 Fax : 040–27847334 email : info@time4education.com website : www.time4education.com AILET2018/10
76. Factual Situation: The defendant, Mr. Dhawan, additional n5,000. He then ran out of money and
wrote to the complainant, Mr. Chaman, with an refused to continue unless payment was made.
offer to sell his house to him for `8,00,000 He MXM Co. engaged another carpenter to complete
promised that he would keep this offer open to him the contract and refused to pay Hasan any further
until Friday. However, on the Thursday Mr. sums. Hasan sued for payment under the original
Dhawan accepted an offer from a third party and agreement and the subsequent agreement. MXM
sold his house. According to Mr. Chaman, he was Co. argued that the agreement to make additional
going to accept this offer but had not said anything payments was unenforceable as Hasan has not
to Mr. Dhawan because he understood that he had provided any consideration to make this
time until Friday. Mr. Dhawan communicated to agreement a valid contract. Decide.
Mr. Chaman that the offer had been withdrawn,
(a) The agreement to pay extra was
through a friend to the complainant. After hearing
unenforceable as Hasan had provided no
this, Mr. Chaman went to find the defendant,
consideration as he was already under an
informing of his acceptance to the offer.
existing contractual duty to complete the work.
Thereafter, the complainant brought an action for
(b) Consideration was provided by Hasan in the
specific performance and breach of contract
form of conferring a benefit on the MXM Co.
against the defendant. Whether the defendant's
by helping them to avoid the penalty clause.
promise to keep the offer open until Friday
Therefore, MXM Co. was liable to make the
morning was a binding contract between the
extra payments promised.
parties and whether he was allowed to revoke this
(c) There was no consideration provided by
offer and sell to a third party?
Hasan as to avoid the penalty clause was the
(a) The statement made by Mr. Dhawan amounts
main object of the contract. Therefore, MXM
to a valid contract and he has committed a
Co. was not liable to make the extra payments
breach by selling the house before Friday.
promised.
(b) The statement made by Mr. Dhawan was
(d) MXM Co. is liable to pay compensation to
nothing more than a promise; there was no
Hasan as they have committed a breach of
binding contract formed. He had
contract by employing another carpenter.
communicated an offer for buying his house to
the complainant and this offer can be revoked Solution:
any time before there is acceptance. As per principle no. 1 the statement of Hassan
(c) The communication by a friend or other party amounts to consideration for extra payment, as he
that an offer had been withdrawn is invalid and was conferring the benefit to MXM company,
could not be treated as if it came from the which was a prevention against the loss of
person himself. liquidated damages, which they have decided with
(d) Promises to keep an offer open until a certain Star Heights Housing Association, so it has value
time is a binding agreement as it is accepted in the eyes of law. Choice (b)
by the other party.
78. Factual Situation: Bournville ran a sales
Solution:
promotion whereby if persons sent in 3 chocolate
As per principle no. 2, Mr. Dhawan revoked his
bar wrappers and a postal order for `100 they
offer before it could be accepted by Mr. Chaman,
would be sent a record. Big Beats owned the
so it was just a promise, and it will not turn into a
copyright in one of the records offered and
contract, as Mr. Chaman never accepted the offer,
disputed the right of Bournville to offer the records
before it’s revocation. Choice (b)
and sought an injunction to prevent the sale of the
77. Factual Situation: MXM Co. is a building records which normally retailed at `1,000. Under
contractor who entered into an agreement with the Copyright Act, retailers are protected from
Star Heights Housing Association to refurbish a breach of copyright if they gave notice to the
block of 27 flats. This contract was subject to a copyright holders of the ordinary retail selling price
liquidated damages clause if they did not complete and paid them 6.25% of this. Bournville gave
the contract on time. The MXM Co. engaged notice stating the ordinary selling price was `100
Hasan to do the carpentry work for an agreed price and three chocolate bar wrappers. The issue is
of `20,000. After six months of commencing the whether the chocolate bar wrappers formed part of
work, Hasan realised he had priced the job too low the consideration?
and would be unable to complete at the originally (a) The wrappers were a mere token or condition
agreed price. He approached MXM Co. who of sale and not consideration.
recognised that the price was particularly low and (b) The wrappers did form part of the
was concerned about completing the contract on consideration for the sale of records despite
time. MXM Co. agreed to make additional the fact that they had no intrinsic economic
payments to Hasan in return for his promise to value in themselves.
carry out his existing obligations. (c) The wrappers did not form part of the
. consideration for the sale of records as they had
MXM Co. agreed to pay Hasan an additional `575 no intrinsic economic value in themselves.
per flat. Hasan continued work on the flats for a (d) There was consideration for the sale of
further period of 6 weeks but only received an records in the form of postal order for `100.
Triumphant Institute of Management Education Pvt. Ltd. (T.I.M.E.) HO: 95B, 2nd Floor, Siddamsetty Complex, Secunderabad – 500 003.
Tel : 040–27898195 Fax : 040–27847334 email : info@time4education.com website : www.time4education.com AILET2018/11
Solution: Solution:
As per principle no. 1, it is clear that chocolate As per principle no. 2, as soon as Tejas kept his
wrappers could be considered as valid money he accepted the offer thereto, so it a valid
consideration, as they could be considered as the contract, which has been accepted consensus –
interest of the company, as they were trying to ad – idem, and cannot be set aside as it was not
promote their company so far, despite the fact having any such option, and Tejas made the
being economic intrinsic, it could be considered as contract after reading the terms of the contract as
valid consideration. Choice (b) per principle 2 and hence he should abide by the
same.
Directions (Q. 79 — Q. 80): Apply the legal principles Choice (c)
to the facts given below and select the most
appropriate answer. 80. Factual Situation: The plaintiffs offered to provide
delivery of a machine tool for a price of `75,535.
Legal Principles: The delivery of the tool was set for 10 months, with
1. Offer is a proposal made by one person to another the condition that orders only qualified as accepted
to do an act or abstain from doing it. The person once the terms in the quotation were met and
who makes the offer is known as the promisor or prevailed over any of the buyer's terms. The buyer
offeror and the person to whom an offer is made is responded to the offer with their own terms and
known as the promisee or the offeree. conditions, which did not include the 'price
2. A contract comes into being by the acceptance of variation clause' listed in the seller's terms. This
an offer. When the person to whom the offer is included a response section which required a
made signifies his consent thereto, the proposal is signature and to be returned in order to accept the
said to be accepted and the parties are at order. The sellers returned this response slip with
consensus ad idem regarding the terms of the a cover letter signaling that delivery would be in
agreement. accordance with their original quotation. The tool
was ready for delivery but the buyers could not
79. Factual Situation: Tejas drove his car to a car accept delivery, for which the sellers increased the
park named Super Car Park (SCP). Outside the price which was in line with their initial terms. This
car park, the prices were displayed and a notice was denied by the buyer and an action was
stated cars were parked at the owner's risk. An brought by the seller to claim the cost of delay and
automatic ticket vending machine provided a interest. Was a contract made with or without a
ticket, a barrier was raised and Tejas parked his price variation clause?
car. In small print on the ticket it was stated that
the ticket is issued subject to conditions displayed (a) The buyer's order was not an acceptance of
on the premises. On a pillar opposite to the the initial offer from the seller but a counter-
machine was a notice stating the owners would not offer which the sellers had accepted by
be liable for any injuries occurring on their returning the signature section of the buyer's
premises. Tejas met with an accident and sought letter and so the contract was completed
damages from SCP. SCP denied any liability on without the price variation clause and
the basis of the exclusion clause which was therefore the seller could not increase the cost
mentioned in the notice on the pillar. Whether of the tool.
there is an offer and acceptance of the exclusion (b) The buyer's order was an acceptance of the
clause? initial offer from the seller and so the contract
was completed with the price variation clause
(a) There is a valid contract between Tejas and and therefore the seller can increase the cost
SCP as SCP had taken reasonable steps to of the tool.
bring exclusion clause to Tejas's attention at (c) The contract was made with price variation
the time of making the contract. clause due to the condition that orders only
(b) The contract was made when Tejas received qualified as accepted once the terms in the
the ticket and parked his car. The ticket quotation were met and prevailed over any of
amounted to a contractual document which the buyer's terms.
effectively referred to the terms which were (d) The contract between the buyer and seller is
clearly visible on the premises. not valid as both the parties are not agreeing
(c) The machine itself constituted the offer. The to the same thing in the same sense.
acceptance was by putting the money into the
machine. The ticket was dispensed after the Solution:
acceptance took place and therefore the As per principle no. 2, the acceptance made by the
exclusion clause was not incorporated into the buyer is not unqualified, so it should be considered
contract. as counter offer, and his counter offer was
(d) The machine itself constituted the offer. The accepted by the company thereto, and it was not
acceptance was by putting the money into the containing the price variation clause, and hence
machine and acceptance of the offer mean price cannot be increased now.
acceptance of all the terms of the offer and Choice (a)
hence SCP is not liable.

Triumphant Institute of Management Education Pvt. Ltd. (T.I.M.E.) HO: 95B, 2nd Floor, Siddamsetty Complex, Secunderabad – 500 003.
Tel : 040–27898195 Fax : 040–27847334 email : info@time4education.com website : www.time4education.com AILET2018/12
81. Legal Principle: Directions (Q. 82 — Q. 83): Apply the legal principles
1. 'Misrepresentation' means and includes -the to the facts given below and select the most
positive assertion, in a manner not warranted appropriate answer.
by the information of the person making it, of
that which is not true, though he believes it to Legal Principles:
be true;any breach of duty which, without an 1. The Tort of Negligence is a legal wrong that is
intent to deceive, gains an advantage of the suffered by someone at the hands of another who
person committing it, or any one claiming fails to take proper care to avoid what a reasonable
under him, by misleading another to his person would regard as a foreseeable risk.
prejudice, or to the prejudice of any one 2. The test of liability requires that the harm must be
claiming under him;causing, however a reasonably foreseeable result of the defendant's
innocently, a party to an agreement, to make conduct, a relationship of proximity must exist and
a mistake as to the substance of the thing it must be fair, just and reasonable to impose
which is the subject of the agreement. liability.
2. The tort of negligent misstatement is defined 3. The claimant must prove that harm would not have
as an inaccurate statement made honestly but occurred 'but for' the negligence of the defendant.
carelessly usually in the form of advice given The claimant must prove, on the balance of
by a party with special skill/knowledge to a probabilities, that the defendant's breach of duty
party that doesn't possess this skill or caused the harm.
knowledge.
82. Factual Situation: Amar worked for an iron works,
Factual Situation: The plaintiff, Mr. Madan, Luxmi Mills & Co. Ltd. operating a remotely
entered into a tenancy agreement with the controlled crane, Amar galvanized items by
defendant, Esso Petroleum, in respect of a petrol dipping them into a large tank of molten metal. In
station owned by the latter. During the course of order to protect its crane operators, whose
the negotiation of the agreement, 'expert' advisers controls were located just a few feet from the tank,
employed by the defendant had provided an Luxmi Mills erected a low wall around the tank and
estimate of the sales which the petrol station could also provided a sheet of corrugated iron that crane
expect which was based on inaccurate information operators placed between themselves and the
and consequently was significantly inflated. The wall. The operators were not facing the tank while
value of the rent on the agreement had been operting the crane. Thus, they could not see the
calculated based on this inflated figure. As a result, operation of the crane and therefore relied upon
it was impossible for the plaintiff to operate the signals from another worker located farther from
petrol station profitably. Whether the plaintiff could the tank. Many other galvanizers at the time
have any action for negligent misrepresentation? situated their operators in enclosed, windowed
(a) The contract could not be held void for spaces from which they could safely see and
misrepresentation as the defendants perform their work. Luxmi Mills eventually adopted
presented the inflated figure as an estimate that practice as well. One day, Amar was working
rather than as a hard fact. on the crane. At one point, he either turned toward
(b) The defendant has no obligation to disclose as the tank or leaned out to see the worker giving him
the parties contracting should obtain the instructions, thereby placing his head outside the
necessary information themselves without iron sheet. A spray of molten metal burned Amar's
relying upon the other party. lip. When it failed to heal and began to ulcerate, he
(c) As the defendant had taken it upon consulted a doctor who diagnosed the wound as
themselves •to employ experts for the purpose cancerous. Amar ultimately died from the spread
of providing an estimate of sales, they owed a of cancer after three years. His widow sued Luxmi
duty of care to the plaintiff to ensure that this Mills for negligence. Whether the employers would
was done on the basis of accurate information. be liable for the full extent of the burn and cancer
Hence, the plaintiff can recover the losses that had developed as a result?
which he had suffered as a result of the (a) The employers are liable for all of the
defendant's negligent misstatement. consequences of their negligence; thus, liable
(d) Both (a) and (c) for the employee's death. His predisposition to
cancer did not matter, nor did the results of the
Solution: injury. The question of liability was, whether
As per facts, the statement made by the party was the defendant could reasonable foresee the
not making any positive assertion rather than it injury.
was based upon the inflated rate, so it would not (b) The employers are not liable because the duty
amount to misrepresentation on the part of Esso of care towards Amar was not breached by
Petroleum, hence the contract cannot be a void them as they were using the same practices
one, but on the other hand as per principle 2 he which were used by other companies at that
was having a duty of care towards, Mr. Madan but time.
as he had not performed it well, he could be sued (c) The employers are not liable because Amar
for the same. suffered injury due to his own negligence in
Choice (d) stepping out of the protective shield.

Triumphant Institute of Management Education Pvt. Ltd. (T.I.M.E.) HO: 95B, 2nd Floor, Siddamsetty Complex, Secunderabad – 500 003.
Tel : 040–27898195 Fax : 040–27847334 email : info@time4education.com website : www.time4education.com AILET2018/13
(d) The employers are liable for burns and not for year 2000 for the students having behavioural and
the death which happened due to Amar's emotional difficulties. The claimants in the instant
cancerous condition which could not have case had resided there between 2000 to 2003,
been known to the employers. being aged 12 to 15 during that time, under the
care of a warden, who was in charge of
Solution: maintaining discipline and the running of the
As per principle no. 1, an employer is liable if he house. The warden lived in the House, with his
cannot provide sufficient amount of care to foresee disabled wife, and together they were the only two
the risk to the employees, and it is clear by the members of staff in the House. His duties were
facts that the employee does not have the risk ensuring order, in making sure the children went
foreseeability. Choice (a) to bed, went to school, engaged in evening
activities, and supervising other staff. It had been
83. Factual Situation: A 13-year-old boy fell from a alleged by some of the boys that the warden had
tree. He went to a hospital where his hip was sexually abused them, including inappropriate
examined, but an incorrect diagnosis was made. advances and taking trips alone with them. A
After 5 days it was found that he was suffering criminal investigation took place some ten years
from avascular necrosis. This was more advanced later, resulting in the warden being sentenced to
and serious than if it had been spotted straight seven years imprisonment. Following this, the
away. Despite receiving treatment, it was victims brought an action for personal injury
determined that he had suffered from a muscular against the employers, alleging that they were
condition (avascular necrosis) which left the boy vicariously liable. Whether the employers of the
with a permanent disability and further left a strong warden may be held vicariously liable for their
probability that he would develop severe employee's intentional sexual abuse of school
osteoarthritis later in life. The expert medical boys placed under his care?
testimony indicated that had his fractured hip been (a) No, vicarious liability could only arise when the
identified on his initial hospital visit, there was a employee is acting during the course of his
25% chance of his condition having been employment and for his employer's benefit.
successfully treated. He is claiming compensation (b) No, the employers cannot be made liable for
for the negligence of hospital. Whether the acts which are not authorised by them.
hospital's negligence on his initial visit had caused (c) Yes, there was a sufficient connection
his injury? between the work that the warden was
(a) No, because there are very less chances that employed to do and the abuse that he
correct diagnosis and treatment would have committed to render it within the scope of
prevented the disability from occurring. employment. The abuse was committed at the
(b) Where there are a number of possible causes, time, premises and during the course of the
the claimant must still prove the defendant's warden's care of the boys.
breach of duty caused the harm or was a (d) Yes, because the employers should be made
material contribution. liable in cases of sexual abuse of differently
(c) Yes, because there are some chances that abled children.
correct diagnosis and treatment would have
prevented the disability from occurring. Solution:
(d) Both (a) and (b) It is clear by the above stated principle that the
Master is liable for the act of his servant done in
Solution: the course of employment, as the warden was
As per above stated principle no. 3, there has to doing all the immoral conduct in the course of
be a certainty that the damage would not have employment for the school, so the owner of the
been incurred until the negligent act has not been school is vicariously liable for the act of the
performed by the party, but here there was every warden. Choice (c)
chance to survive, with this condition, so option(a)
may prevail but again after following the 2nd part of 85. In 2017, Special Leave Petition for re-investigation
the principle 3, we may see that we need to prove in the Mahatma Gandhi murder case was filed by
the negligence of the party without reasonable ______.
doubt, in case of balance of probability so (a) Amrendra Sharan
option(b) shows the Hospital’s negligence for (b) Pankaj K. Phadnis
damage contribution, hence both the options (c) Tushar Gandhi
should prevail. Choice (d) (d) Raju Ramachandran

84. Legal Principle: Vicarious liability is when Solution:


employers are held liable for the torts committed The petition filed by the Mumbai-based Dr Pankaj
by their employees during the course of Phadnis , a researcher and a trustee of Abhinav
employment.
Bharat organization, in front of a bench comprising
Factual Situation: New Vision School opened a of Justices S A Bobde and L Nageswara Rao.
boarding house (Shivaji House) for boys in the Choice (b)

Triumphant Institute of Management Education Pvt. Ltd. (T.I.M.E.) HO: 95B, 2nd Floor, Siddamsetty Complex, Secunderabad – 500 003.
Tel : 040–27898195 Fax : 040–27847334 email : info@time4education.com website : www.time4education.com AILET2018/14
86. Who is the first Indian woman to be elected as the 91. Which State Government, in India, has recently
Judge of the Intemational Tribunal for the Law of sanctioned stipend for junior lawyers practicing in
Sea? the courts?
(a) Dr. Neeru Chadha (a) Delhi (b) Punjab
(b) Justice Gita Mittal (c) Kerala (d) Maharashtra
(c) Vijaya Lakshmi Pandit
(d) Dr. Menaka Guruswamy Solution:
The Junior lawyers having earning less than `1
Solution: lakh/annum, aged less than 30 years, and having
Ms. Neeru Chadha has been appointed as the first less than 3 years practicing experience will get the
woman judge from India, as a judge for stipend of `5,000/- per month.
International Tribunal from the law of the sea, she Choice (c)
has been appointed for a 9 years’ term from 2017
– 2026 by getting the highest of 120 votes in Asia 92. From the field of Law and Justice, who is awarded
pacific group. Choice (a) the Nari Shakti Puraskar 2018 on the International
Women's Day?
87. Which of the following river(s) is/are declared as a (a) Geeta Mittal
legal entity by an Act of Parliament? (b) Sudha Bharadwaj
(a) Ganga (b) Whanganui (c) Flavia Agnes
(c) Yamuna (d) All of the above (d) Indira Jaisingh

Solution: Solution:
River Whanganui got the status like a legal entity Acting Chief justice of the Delhi High Court Geeta
as per the recent law passed by the New Zealand Mittal, was the awardee of Nari Shakti Puraskaar,
government, where river Ganga and Yamuna are 2018. It is “highest civilian honour for women in
not considered as legal entity as Supreme Court India”. Choice (a)
overruled the judgement of Uttarakhand High
Court. Choice (b) 93. Recently, the Supreme Court allowed ______
euthanasia and right to give advance medical
88. Which of the following lawyers approached the directives ______, stating that human beings have
Supreme Court challenging the existing system of the right to die with dignity as part of fundamental
'designation of Senior Advocates'? right to life.
(a) Meenakashi Lekhi (a) active, dying will
(b) Vrinda Grover (b) passive, living will
(c) Indira Jaisingh (c) active, living will
(d) Meenakshi Arora (d) passive, dying will

Answer key: Answer key:


Choice (c) Choice (b)

89. 'Drunken Driving' is punishable under _______ 94. Local self-governance is an example of
(a) Indian Penal Code (a) federalism
(b) Motor Vehicles Act (b) democratic decentralisation
(c) Road Safety Standards Act (c) direct democracy
(d) National Highways Act (d) administrative delegation

Solution: Answer key:


Sec. 185 of the Motor Vehicle Act, 1988 states that Choice (b)
a person who is driving and having alcohol
exceeding 30mg/100ml in his blood, shall be 95. The institutional infrastructure under the
liable. Choice (b) Insolvency and Bankruptcy Code, 2016, rests on
certain pillars:
90. The Supreme Court on 12 December 2017, had I. Insolvency Professionals
directed that _______special criminal courts, to be II. Information Utilities
set up to exclusively deal with cases involving III. Adjudication
_______should start functioning from March 1, IV. The Insolvency and Bankruptcy Board of India
2018. V. Asset Construction Companies
(a) 12, MPs/MLAs
(b) 12, Rape Accused Correct code is:
(c) 24, child-sexual abuser (a) I, II and III (b) II, III and IV
(d) 24, Terrorism related offences (c) I, II, II and IV (d) All of the above

Answer key: Answer key:


Choice (a) Choice (c)
Triumphant Institute of Management Education Pvt. Ltd. (T.I.M.E.) HO: 95B, 2 Floor, Siddamsetty Complex, Secunderabad – 500 003.
nd

Tel : 040–27898195 Fax : 040–27847334 email : info@time4education.com website : www.time4education.com AILET2018/15


96. In 2017, the Supreme Court held that right to Solution:
privacy is protected under Article 21 of the In the case of Shyra bano v UOI, the Supreme
Constitution of India in the context of ______. court by a 5-judge bench decided to declare
(a) State Surveillance instant triple talaq illegal and criminalize it with
(b) Power of search & seizure the punishment of 3 years jail and a fine,
(c) Homosexuality considering it a cognizable and non – bailable
(d) Indian Biometric Identification Scheme offence, and passed by the parliament.
Choice (a)
Solution:
In the case of Justice K. S. Puttaswamy (Retd.) 101. In August 2017, the Union Ministry of Law &
and Anr. vs Union of India And Ors., Supreme Justice in association with the National Legal
Court stated that Indian Biometric identification Service Authority (NALSA) launched 'Tele-Law'
scheme should not be mandatory as it could be scheme in to provide legal aid services to
violation of article 21, right to privacy in right to life. marginalised communities and citizens living in
Choice (d) rural areas through digital technology.
(a) Andhra Pradesh (b) Bihar
97. The age of consent for sexual intercourse between (c) Madhya Pradesh (d) Rajasthan
a husband and wife has been made ______ from
______ years by the Supreme Court of India. Answer key:
(a) 18, 16 (b) 16, 15 (c) 21, 18 (d) 18, 15 Choice (b)
Solution: 102. In March 2018, the Supreme Court of India has
As per sec. 375 (2) of the Indian Penal Code, of held that there cannot be a stay of more than
the age of a female who is married, is 15 years, ______on trial of______
sexual intercourse would not constitute rape, and (a) 6 months, civil and criminal cases
the age of male for sexual intercourse as husband (b) 6 months, criminal cases
is 18 years. Choice (d) (c) 3 months, rape cases
(d) 3 months, civil and criminal cases
98. An accused is entitled to statutory bail (default bail)
if the police failed to file the charge-sheet within Answer key:
______ of his arrest for the offence punishable Choice (a)
with 'imprisonment up to 10 years'.
(a) 30 days (b) 60 days 103. Which country has withdrawn from the
(c) 90 days (d) 180 days International Criminal Court (ICC) citing reasons
of international bias in March, 2018?
Solution:
(a) Burundi (b) Russia
under Section 167(2)(a)(2) of Code of Criminal
(c) South Africa (d) Philippines
procedure, if the police failed to file the charge-
sheet within 60 days of his arrest for the offence Answer key:
punishable with ‘imprisonment up to 10 years. Choice (d)
Choice (b)
104. Regarding foreign law firms, the Supreme Court
99. Nelson Mandela Rules deals with _______ held
(a) Apartheid I. That foreign law firms can set up offices in
(b) Truth and Reconciliation India.
(c) Prison Management II. Foreign lawyers can practice in India.
(d) Global Peace-making III. Foreign lawyers can visit India for a
temporary period on a fly in and fly out basis.
Answer key:
IV. Foreign lawyers can give advice to their
Choice (c)
clients on Indian laws.
100. The Lok Sabha passed the Muslim Women (a) I, II, III (b) II, IV (c) III (d) III, IV
(Protection of Rights on Marriages) Bill 2017. Answer key:
Which of the following are true about it? Choice (c)
I. It declares instant triple talaq illegal and
criminalizes it. 105. Which is the world's first country to enact a law
II. It makes declaration of talaq a bailable offence. under which companies and government
III. A husband declaring talaq can be imprisoned agencies that employ at least 25 people will be
for up to two years along with a fine. fined if they can't prove they pay men and women
IV. It entitles Muslim woman against whom triple equally?
talaq has been declared to seek subsistence (a) Iceland (b) India
allowance from her husband for herself and (c) Sweden (d) France
for her dependent children.
(a) I and IV (b) I, III and IV Answer key:
(c) I, II and IV (d) All of the above Choice (a)
Triumphant Institute of Management Education Pvt. Ltd. (T.I.M.E.) HO: 95B, 2 Floor, Siddamsetty Complex, Secunderabad – 500 003.
nd

Tel : 040–27898195 Fax : 040–27847334 email : info@time4education.com website : www.time4education.com AILET2018/16


SECTION – D
REASONING
Directions (Q. 106 — Q. 110): Read the following 108. Akash is second to the right of Esha is true.
information carefully and answer the questions given Choice (d)
below.
109. Hema is sitting to the immediate left of Esha.
Eight friends Akash, Bunty, Chandan, Deepak, Esha, Choice (b)
Fiza, Gazal and Hema are sitting around a circle facing
the centre. Deepak is between Akash and Esha only, 110. Esha is third to the right of Gazal. Choice (d)
Fiza is second to the left of Esha. Bunty is between
Chandan and Gazal only. Fiza is between Gazal and 111. Harsh moves 15 kms in East direction, then turns
Hema only. towards North and moves 4 kms. From there he
turns West and travels 12 kms. How far and in
106. Who is fourth to the right of Bunty? which direction is he from his starting point?
(a) Esha (b) Hema (a) 5 kms, North-East
(c) Gazal (d) Deepak (b) 5 kms, South-West
(c) 27 kms, North-East
107. In which of the following pairs, second person is (d) 19 kms, North-East
to the immediate left of the first person?
(a) Deepak, Akash (b) Bunty,Gazal Solution:
(c) Bunty, Chandan (d) Hema, Esha The line diagram is as follows:
12
108. Which of the following is definitely true?
(a) Chandan is to the immediate right of Bunty.
<
(b) Esha is to the immediate left of Chandan.
(c) Deepak is second to the left of Hema.
(d) Akash is second to the right of Esha. ^4
3
109. Who is sitting in the immediate left of Esha? • >
(a) Fiza (b) Hema (c) Gazal (d) Deepak Harsh
15
110. Who is third to the right of Gaza'? The distance between his starting point and
(a) Fiza (b) Akash (c) Hema (d) Esha ending point is √32 + 42 = 5km, in north-east
direction. Choice (a)
Solution for questions 106 to 110:
112. Pratham travels 10 metres in North direction, he
Fiza is second to the left of Esha. Now, Deepak is
turns right and travels 20 metres. Again, he turns
sitting between Akash and Esha only. Therefore,
towards right and travels 25 metres. Then, he
Deepak sits to the immediate right of Esha and Akash
turns towards left and travels 15 metres. In which
sits to the right of Deepak. Fiza is between Gazal and
direction and how far is Pratham from his original
Hema only. Gazal and Hema sits to the immediate left
position?
and to the immediate right of Fiza in any order. But
(a) 35 metres, East
Bunty is sitting between Chandan and Gazal only. So,
(b) 38.07 metres, South-East
gazal will sit to the immediate left of Fiza and Chandan
(c) 25.08 metres, West
will seat to the immediate right of Akash. The final
(d) 25 metres, North-East
arrangement is as follows:
Solution:
Akash
The line diagram is as follows:
Deepak
Chandan 20
>
10
Bunty Esha ^
v 25
15
Gazal Hema
>
Fizza
The distance between his starting point and
106. Esha is fourth to the right of Bunty. Choice (a) ending point is √352 + 152 = 38.07m, in south-
east direction.
107. Choice (c) is correct. Choice (c) Choice (b)
Triumphant Institute of Management Education Pvt. Ltd. (T.I.M.E.) HO: 95B, 2nd Floor, Siddamsetty Complex, Secunderabad – 500 003.
Tel : 040–27898195 Fax : 040–27847334 email : info@time4education.com website : www.time4education.com AILET2018/17
Directions (Q. 113 — Q. 119): Read the following Spielberg film.
information carefully and answer the questions given (d) Exactly three film buffs see the Stanley
below. Kubrick film.

Seven film buffs — Aamna, Archana, Aradhana, Anu, 115. Which one of the following could be a complete
Aahana, Anuja and Aashna — attend a showing of and accurate list of the film buffs who do Not see
classic movies. Three films are shown, one directed by the Stanley Kubrick film?
Satyajit Ray, one by Stanley Kubrick, and one by (a) Aamna, Aahana
Steven Spielberg. Each of the film buff see only one of (b) Aamna, Archana, Aahana
the three films. The films are shown only once, one film (c) Aamna, Anu, Aashna
at a time. The following restrictions must apply: (d) Aamna, Anuja, Aashna
i. Exactly twice as many of the film buffs see the
Stanley Kubrick film as the Satyajit Ray film. 116. If exactly one film buff sees the Steven Spielberg
ii. Aamna and Aahana do not see the same film as film, then which one of the following must be
each other. true?
iii. Archana and Anu do not see the same film as each (a) Aamna sees the Satyajit Ray film.
other. (b) Anu sees the Satyajit Ray film.
iv. Anuja and Aashna see the same film as each (c) Anuja sees the Stanley Kubrick film.
other. (d) Archana sees the Satyajit Ray film.
v. Aradhana sees the Stanley Kubrick film.
vi. Aamna sees either the Satyajit Ray film or the 117. Which of the following must be true?
Steven Spielberg film. (a) Archana, Aradhana and Anuja do not all see
the same film.
113. Which one of the following could be an accurate (b) Archana sees a different film than Anuja
matching of film buffs to films? does.
(a) Aamna: the Steven Spielberg film; Archana: (c) Aamna sees a different film than Archana
the Satyajit Ray film; Anuja: the Satyajit Ray does.
film (d) Aamna, Aradhana and Anu do not all see the
(b) Anu: the Steven Spielberg film; Aahana: the same film.
Steven Spielberg film; Anuja: the Steven
Spielbergfilm 118. If Anuja sees the same film as Aamna does, then
(c) Anu: the Stanley Kubrick film; Aahana: the which one of the following could be true?
Stanley Kubrick film; Aashna: the Stanley (a) Aahana sees the Stanley Kubrick film.
Kubrickfilm (b) Aashna sees the Satyajit Ray film.
(d) Archana: the Stanley Kubrick film; Aahana: (c) Archana sees the Stanley Kubrick film.
the Steven Spielberg film; Anuja: the Satyajit (d) Aamna sees the Satyajit Ray film.
Ray film
119. Each of the following could be a complete and
114. Each of the following must be false EXCEPT accurate list of the film buffs who see the Satyajit
(a) Aahana is the only film buff to see the Ray film EXCEPT
Satyajit Ray film. (a) Aamna, Archana
(b) Aahana is the only film buff to see Stanley (b) Anuja, Aashna
Kubrick film. (C) Anu, Aahana
(c) Aashna is the only film buff to see the Steven (d) Aamna, Anu

Solution for questions 113 to 119:

Let us denote the three directors as SR, SK and SS (as per their first letters of their names).

From (I), there will be two cases:

case(i): only one buff will watch the movie of SR and only two buffs will watch the movie of SK.

case(ii): only two buffs will watch the movie of SR and only four buffs will watch the movie of SK.

Let us consider case(i):

As Aradhana watches the movie by SK. So, only one more buff will watch his movie. Therefore, from (iv), Anuja
and Aashna watch the movie by SS. From (vi), Aamna sees either the SR film or the SS film. If Aamna watches the
SR film, Archana and Anu will watch the SK film and the SS film in any order. And Aahana watches the SS film. If
Aamna watches the SS film, Aahana will watch either the SK film or the SR film. If Aahana watches the SR film,
Archana and Anu watches the SK film and the SS film in any order. If Aahana watches the SK film, Archana and
Anu watches the SK film and the SR film in any order. So, the sub-cases will be as follows:

Triumphant Institute of Management Education Pvt. Ltd. (T.I.M.E.) HO: 95B, 2nd Floor, Siddamsetty Complex, Secunderabad – 500 003.
Tel : 040–27898195 Fax : 040–27847334 email : info@time4education.com website : www.time4education.com AILET2018/18
case(ia) case(ib) case(ic)
SR SK SS SR SK SS SR SK SS
Anu/
Aamna Aradhana Anuja Aradhana Anuja Aahana Aradhana Anuja
Archana
Archana/ Anu/
Aashna Aahana Aashna Aashna
Anu Archana
Aahana Aamna Aamna
Anu/ Archna/ Archna/
Archana Anu Anu

Now, let us consider case (ii):

Both Anuja and Aashna can watch either the SR film or the SK film. If Anuja and Aashna watch the SR film, Aamna
watches the SS film. Then, the three buffs who watch the SK film are Archana, Anu and Aahana. But, Archana and
Anu cannot watch the same film. Hence, Anuja and Aashna watch the SK film. Aamna can watch either the SR film
or the SS film. If Aamna watches the SR film, Aahana watch either the SK film or the SS film. If Aahana watches
the SK film, Anu and Archana watch the SR film and the SS film in any order. If Aamna watches the SS film, Anu
and Archana watch the SK film and the SR film in any order. If Aamna watches the SS film, Archana and Anu watch
the SK film and the SR film in any order. The sub-cases are as follows:
case(iia) case(iib) case(iic)
SR SK SS SR SK SS SR SK SS
Anu/
Aamna Aradhana Aamna Aradhana Aahana Aahana Aradhana Aamna
Archana
Archana/ Archana/ Anu/
Anuja Anuja Anuja
Anu Anu Archana
Aashna Aashna Aashna
Anu/ Archana/
Aahana
Archana Anu

113. Choice (b) is the accurate matching of the film buffs to films, as in case(ia). Choice (b)

114. Choice (a) is true as in case(iib). Choice (a)

115. From cases (iib) and (iic), we get that choice (b) is possible. Choice (b)

116. In case(ii) we can see that only one buff watches the SS film and Anuja definitely watches the SK film.
Choice (c)

117. Choice (d) is always true. Choice (d)

118. From cases (ib) and (ic), choice (a) and choice (c) are possible answers. While the official key is (c),
choice (a) is also possible as shown in (ib). Choice (a, c)

119. Anuja and Aashna can never see the SR film together. Choice (b)

Directions (Q. 120 — Q. 121): In each of the following


questions, there are two statements labelled as Solution:
Assertion (S) and Reason (R). Both S and R are false. Choice (d)
Give answer 121. Assertion (S) — Moon cannot be used as a
(a) if both 'S'& 'R' are true and 'R' is the correct satellite for communication.
explanation of 'S'. Reason (R) — Moon does not move in the
(b) if 'S' is true but 'R' is false equatorial plane of the Earth.
(c) if 'S' is false and 'R' is true
(d) if both 'S' &'R' are false Solution:
Both S and R are true and R is one of the reasons
120. Assertion (S) — Bulb filament is made of for S.
titanium. Choice (a)
Reason (R) — The filament should have low
melting point.
Triumphant Institute of Management Education Pvt. Ltd. (T.I.M.E.) HO: 95B, 2nd Floor, Siddamsetty Complex, Secunderabad – 500 003.
Tel : 040–27898195 Fax : 040–27847334 email : info@time4education.com website : www.time4education.com AILET2018/19
Directions (Q. 122 — Q. 126): Given below is a 123. When a client reports their involvement in a
passage followed by few statements about that serious crime, their Mediators is legally obliged to
passage. You must select one of the following report this?
answers: (a) Definitely True
(b) Probably True
Definitely True: The statement follows logically from (c) Definitely False
the information contained in the passage. (d) Data Inadequate

Definitely False: The statement is logically false from Solution:


the information contained in the passage. The paragraph says, in cast of serious crime,
breach of confidentiality “may be” considered
Probably False: The statement is more likely to be legitimate. This indicates that there is no legal
false than true, but not definitely false beyond a obligation. Hence, it is definitely false.
reasonable doubt, based solely on the information in Choice (c)
the passage.
124. Mediators have some flexibility regarding what
Data Inadequate: It is not possible to determine they deem serious enough to lead them to breach
whether the statement is true or false without further confidentiality.
information. (a) Definitely True
(b) Definitely False
Mediators recognize that the law is generally in place (c) Probably False
to uphold mediator - client confidentiality, however, (d) Data Inadequate
there are situations that may occur where the mediator
is under obligation to break that confidence. This Solution:
obligation can vary depending upon where the This statement is definitely true according to the
mediator is practicing and it may occur as a result of given passage.
their employment contract or of the law. Where such Choice (a)
an issue does occur, the mediator is expected to firstly
try and discuss the presenting issue with their client; 125. Most Mediators agree with the conditions that the
however, in situations where the factors under law places on them relating to breaching client
consideration are particularly urgent, it is accepted that confidentiality.
this cannot always be the case. (a) Definitely True
(b) Definitely False
Legitimate breaches of confidentiality relate to (c) Probably False
circumstances where the information the client has (d) Data Inadequate
shared relates to anti-national activities; information of
this nature must be reported. There are other Solution:
circumstances where breaching confidentiality may be According to the statement, most mediators
considered legitimate, for example, in the case of agree with the conditions that the law places on
serious crime or suspected child abuse. Mediation them relating to breaching client confidentiality.
service providers and mediators practicing But according to the passage we cannot
independently have their own boundaries but must conclude that this is actually being done in
agree this contractually with their client at the outset of practice.
the client-counsel relationship. Choice (d)

122. Being obliged to break confidentiality in a 126. If a mediator breaches his client's confidence
mediator-client relationship is always a direct about a serious issue without first informing their
result of the law. client of their intentions, they are breaking the
(a) Definitely True law.
(a) Definitely True
(b) Definitely False
(b) Probably False
(c) Data Inadequate
(c) Definitely False
(d) Probably False (d) Data Inadequate

Solution: Solution:
According to the statement, being obliged to According to the statement, if a mediator
break confidentiality in a mediator-client breaches his client's confidence about a serious
relationship is always a direct result of the law. issue without first informing their client of their
But in the given passage, this obligation can vary intentions, they are breaking the law. But
depending upon where the mediator is practicing according to the passage a mediator need to
and it may occur as a result of their employment discuss with the client, whenever there is a
contract or of the law. serious issue. Hence, the given statement is
Hence, this is definitely false. definitely false.
Choice (b) Choice (c)

Triumphant Institute of Management Education Pvt. Ltd. (T.I.M.E.) HO: 95B, 2nd Floor, Siddamsetty Complex, Secunderabad – 500 003.
Tel : 040–27898195 Fax : 040–27847334 email : info@time4education.com website : www.time4education.com AILET2018/20
Directions (Q. 127 — Q. 128): Find out the wrong 132. If in a certain code language 'PERFECT' is
number in the series. written as '1 1 6', then how will 'DIVORCE' be
written in that code?
127. 1, 2, 8, 33, 148, 760, 4626 (a) 111 (b) 120 (c) 113 (d) 117
(a) 760 (b) 148 (c) 4626 (d) 2
Solution:
Solution: The logic is to add the place value of the opposite
The correct series is as follows: letters of the given word. Hence, the code for
'DIVORCE' will be 23 + 18 + 5 + 12 + 9 + 24 + 22
1 2 8 33 148 765 4626 = 113.
× 1 + 1 × 2 + 4 × 3 + 9 × 4 + 16 × 5 + 25 × 6 + 36 Choice (c)

Hence, 760 is the wrong number in the series. 133. Which is the largest fraction?
Choice (a) (a) 3/4 (b) 7/8 (c) 4/5 (d) 7/9

128. 888, 440, 216, 104, 48, 22, 6 Solution:


(a) 440 (b) 216 (c) 22 (d) 6 3/4 = 0.75, 7/8 = 0.875, 4/5 = 0.80, 7/9 = 0.777.
Hence, 7/8 is the largest fraction.
Solution: Choice (b)
The correct series is as follows:
134. How many degrees are there between clock
888 440 216 104 48 20 6 hands at 3.15?
2–4 2–4 2–4 2–4 2–4 2–4
(a) 0 (b) 5.5 (c) 7.5 (d) 10
Hence, 22 is the wrong number in the series.
Choice (c) Solution:
11
 = |30H − M|
2
Directions (Q. 129 — Q. 130): Find the one that does 11
 = |30 × 3 − × 15|
not belong to the group. 2
= |90 − 82.5|
129. (a) 1 : 0 (b) 7 : 50 (c) 6 : 35 (d) 3 : 8 = 7.5°
Choice (c)
Solution:
We can see that (a), (c) and (d) are in the form Directions (Q. 135 — Q. 140): Read the following
n : n ^ 2 – 1. But (b) is of the form n : n ^ 2 + 1. short passages and choose the best answer to the
Choice (b) questions that follow each passage.

130. (a) LO (b) HS (c) DW (d) JR 135. Modern science is built on the process of posing
hypotheses and testing them against
Solution: observations — in essence, attempting to show
(1): As there is only one pair of letters where a that the hypotheses are incorrect. Nothing brings
vowel(LO) is present it will be the pair which does more recognition than overthrowing conventional
not belong to the group. Choice (a) wisdom. It is accordingly not surprising that some
(2): (a), (b) and (c) are opposite pairs, while (d) is scientists are sceptical of the widely accepted
not an opposite pair. Choice (d) predictions of global warming. What is instead
remarkable is that with hundreds of researchers
131. If in a certain code language 'BETTER' is written striving to make breakthroughs in climatology,
as 'EHWQBO', then how will 'LAWYER' be very few find evidence that global warming is
written in that language? unlikely.
(a) ODZVBO (b) OBZVDO
The information above provides the most support
(c) DOZVBO (d) OVZOBD
for which one of the following statements?
Solution: (a) Most researchers in climatology have
substantial motive to find evidence that
B E T T E R would discredit the global warming
+3 +3 +3 –3 –3 –3 hypothesis.
(b) There is evidence that conclusively shows
E H W Q B O that the global warming hypothesis is true.
(c) Most scientists who are reluctant to accept
Similarly, the code for LAWYER will be:
the global warming hypothesis are not acting
L A W Y E R in accordance with the accepted standards
+3 +3 +3 –3 –3 –3 of scientific debate.
(d) Research in global warming is primarily
O D Z V B O driven by a desire for recognition in the
Choice (a) scientific community.
Triumphant Institute of Management Education Pvt. Ltd. (T.I.M.E.) HO: 95B, 2nd Floor, Siddamsetty Complex, Secunderabad – 500 003.
Tel : 040–27898195 Fax : 040–27847334 email : info@time4education.com website : www.time4education.com AILET2018/21
Solution: 137. Arbitrator: The shipping manager admits that he
According to the passage, a researcher gets decided to close the old facility on October 14
more recognition when he disproves something and to schedule the new facility's opening for
which is already accepted as truth. Therefore, October 17, the following Monday. But he also
some scientists are skeptical of the widely claims that he is not responsible for the business
accepted predictions of global warming. Hence, that was lost due to new facility's failing to open
they are trying to prove it wrong but a few as scheduled. He blames the contractor for not
scientists are getting evidence that the widely finishing on time, but he too, is to blame, for he
accepted prediction of global warming is false. was aware of the contractor's typical delays and
Therefore, only (a) is the correct option. should have planned for this contingency.
Choice (a)
Which one of the following principles underlies
136. The indigenous people of Tago are clearly the arbitrator's argument?
related to the indigenous people of Taminia, but (a) A manager should seeto it that contractors
were separated from them when the land bridge do their job promptly.
between Taminia and Tago disappeared (b) A manager should be held responsible for
approximately 10,000 years ago. Two thousand mistakes by those whom the manager
years after the disappearance of the land bridge, directly supervises.
however, there were major differences between (c) A manager should take foreseeable
the culture and technology of the indigenous problems into account when making
Tagoians and those of the indigenous Taminians. decisions.
The indigenous Tagoian unlike their Taminian (d) A manager, and only a manager, should be
relatives, had no domesticated dogs, fishing held responsible for a project's failure.
nets, polished stone tools, or hunting implements
like the boomerang and the spear-thrower. Solution:
Each of the following, if true, would contribute to The arbitrator is blaming both the manager and
an explanation of differences described above the contractor for delaying the opening of the new
EXCEPT facility. The manager did not take into
(a) Although the technological and cultural consideration of the fact that the contractor
innovations were developed in Taminia more usually make these type of delays. He is blaming
than 10,000 years ago, they were developed the contractor as he did not finish the work on
by groups in Northern Taminia with whom time. Option (c), underlies the arbitrator's
the indigenous Tagoian had no contact prior argument where he/she wants to state that, since
to the disappearance of the land bridge. the manager knew about the problems with the
(b) Devices such as the spear-thrower and the contractor, he/she should have taken into
boomerang were developed by the consideration while deciding to close the old
indigenous Tagoian more than 10,000 years facility. Choice (c)
ago.
(c) After the disappearance of the land bridge the 138. Anita: Television programs and movies that
indigenous Tagoian simply abandoned certain depict violence among teenagers are extremely
practices and technologies that they had popular. Given how influential these media are,
originally shared with their Taminian relatives. we have good reason to believe that these
(d) Indigenous people of Taminia developed depictions cause young people to engage in
hunting implements like boomerang and the violent behaviour. Hence, depictions of violence
spear-thrower after the disappearance of the among teenagers should be prohibited from
land bridge. movies and television programs.
Amrita : But you are recommending nothing short
Solution: of censorship! Besides which, your claim that
In (a), it is clearly states that the technological television and movie depictions of violence
and cultural innovations were developed by the causes violence is mistaken: violence among
Northern Taminians, who do not have any contact young people predates movies and television by
with the Tagoians. So, there is clearly a difference. centuries.
In (b), it is stating that the spear-thrower and the
boomerang were developed by the Tagoian more Amrita's attempted refutation of Anita's argument
than 10,000 years ago. Since, there was a is vulnerable to criticism on which one of the
difference between the two groups, therefore, the following grounds?
Taminians could have the weapons with them. In (a) It confuses a subjective judgment of private
(c), the difference is clearly stated, as it is moral permissibility with an objective
mentioned that the Tagoian abandoned certain description of social fact.
practices and technologies that they had (b) It presupposes that an unpopular policy
originally shared with their Taminian relatives. In cannot possibly achieve its intended
(d), the difference is clearly mentioned as it is purpose.
stated that the weapons were developed after the (c) It cite purported historical facts that cannot
disappearance of the land bridge. Choice (b) possibly be verified.
Triumphant Institute of Management Education Pvt. Ltd. (T.I.M.E.) HO: 95B, 2nd Floor, Siddamsetty Complex, Secunderabad – 500 003.
Tel : 040–27898195 Fax : 040–27847334 email : info@time4education.com website : www.time4education.com AILET2018/22
(d) It rules out something as a cause of a current rate at which the information is delivered is not
phenomenon solely on the ground that the the reason for the confusion. Hence, the
phenomenon used to occur without that assumption of the author to come to the
thing. conclusion is (a), where the number of broadcast
news stories to which a person is exposed in not
Solution: a factor. Choice (a)
Anita's argument is that, television programs and
movies should be prohibited among the 140. Café's Coffee Emporium stocks only two
teenagers which depict violence, as this media decaffeinated coffees: English Roast and
is quite influential and is the main reason that German Mocha. Ratan only serves decaffeinated
these depictions are causing young people to coffee, and the coffee he served after dinner last
engage in violent behavior. Amrita refuted the night was far too smooth and mellow to have
argument given by Anita by saying that television been English Roast. So, if Ratan still gets all his
and movies that depicts violence cannot be the coffee from Café's, what he served last night was
reason for the violence that are caused by German Mocha.
youngsters, as violence among young people
was also there when there was no media. The The argument above is most similar in its logical
flaw in Amrita's argument is that as Amrita is structure to which one of the following?
saying that the media cannot be the reason for (a) Sunny wants to take three friends to the
violence among youngsters as violence among beach. His mother owns both a sedan and a
youngsters was already there before the arrival convertible. The convertible can
of media, but she is not talking about the scale at accommodate four people. Although the
which violence was occurring and Anita is talking sedan has a more powerful engine, if Sunny
about the scale of violence among youngsters. borrows a vehicle from his mother, he will
So, option (d) is the correct answer. Choice (d) borrow the convertible.
(b) Ruchi can either take a two-week vacation in
139. Most people feel that they are being confused by July or wait until October and take a three-
the information from the broadcast news. This week vacation. The trail she had planned to
could be the effect of the information's being hike requires three weeks to complete but is
delivered too quickly or of its being poorly closed by October, so if Ruchi takes a
organized. Analysis of the information content of vacation, it will not be the one she had
a typical broadcast news story shows that news planned.
stories are far lower in information density than (c) If Tamanna does not fire her assistant, her
the maximum information density with which staff will rebel and her department's
most people can cope at any one point of time. efficiency will decline. Losing her assistant
So, the information in typical broadcast news would also reduce its efficiency, so, if no
stories is poorly organised. alternative solution can be found, Tamanna's
department will become less efficient.
Which one of the following is an assumption that (d) Wikro, Inc. has offered Arshit a choice
the argument requires in order for its conclusion between a job in sales and a job in research.
to be properly drawn? Arshit would like to work at Wikro but he
(a) It is not the number of broadcast news would never take a job in sales when another
stories to which a person is exposed that is job is available, so if he accepts one of these
the source of the feeling of confusion. jobs, it will be the one in research.
(b) Poor organisation of information in a news
story makes it impossible to understand the Solution:
information. In the given passage there are only two options
(c) Being exposed to more broadcast news available: (i) English Roast and (ii) German
stories within a given day would help a Mocha. Since, the taste of the coffee that Ratan
person to better understand the news. sells cannot be matched with English Roast, so,
(d) Most people can cope with a very high it has to be German Mocha.
information density. In (a), it is not telling why he didn't choose the
sedan. In (b), Ruchi does not have a choice of
Solution: her own. Whatever option she choose she will
As per the given passage, people are getting never end up in going to a vacation which she
confused by the information from the broadcast planned. In (c), whatever decision tamanna
news because of either the information being takes, it will end up in reducing her department's
delivered too quickly or its being poorly efficience. In (d), Arshit has two options: it can be
organized. But the author says that since the rate either a job in sales or a job in research. Arshit is
at which information is delivered to us is quite low interested in research. So, if at all Arshit will
than a human being's capacity, so, the reason for accept a job in Wikro, he will choose a job in
the confusion is not in the delivery part. Hence, research, which is in sync with the given
the reason is that the information is poorly passage. Hence, (d) is our required option.
organized. But the author didn't prove why the Choice (d)

Triumphant Institute of Management Education Pvt. Ltd. (T.I.M.E.) HO: 95B, 2nd Floor, Siddamsetty Complex, Secunderabad – 500 003.
Tel : 040–27898195 Fax : 040–27847334 email : info@time4education.com website : www.time4education.com AILET2018/23
SECTION – E
MATHEMATICS
141. Two cards are drawn together from a pack of 52 Given the inner surface area of the rectangular
cards. The probability that one is a club and one hall = 1200 sqm.
is a diamond, is a club and one is a diamond, is (lh + bh) = 1200
(a) 13/102 (b) 46/104 ∵ h = 7.5 m
(c) 12/21 (d) 13/200 1200
l+b=
15
Solution: l +b =80 ---------- (2)
In a pack of 52 cards, there will be 13 club cards, Therefore verifying from the options, the only
13 diamond cards. possible values of length and breadth of the hall
The probability that one card is club and the which satisfy (1) and (2) are 50 m and 30 m
other is diamond when two cards are drawn respectively. Choice (b)
(13C1 )((13C1 )
= 144. The ratio between the length and the breadth of
(52C1 )
(13)(13) a rectangular Jogging Park is 3 : 2. If Ashish
= 52 × 51 cycling along the boundary of the park at the
2 speed of 12 km/hr completes one round in 8
13 minutes, then the area of the park (in sq. m) is
= Choice (a) (a) 3,07,200 sq. m (b) 30,720 sq. m
102
(c) 1,53,600 sq. m (d) 15,360 sq. m
142. Sanju has two watches with a 12-hour cycle. One
of these watches, gains one minute a day and the Solution:
other loses 1 and 1/2 minutes per day. If Sanju Ratio between the length and the breadth of a
sets both the watches at the correct time, how rectangular park = 3 : 2
long will it be before they again show the correct Let the length be 3x and the breadth be 2x.
time together? Ashish is covering a distance of 10x km in 8
(a) 482 days (b) 290 days minutes at a speed of 12 km/hr
(c) 1440 days (d) 730 days 8
 12 km/hr x hr = 10x km
Solution:
60
x = 0.16 km
Let C1 be the watch that gains 1 minute per day x = 160 m
1
C2 be the watch that loses 1 and minute per day Length = 480 m
2 Breadth = 320 m
∵ The two watches are of 12-hour cycle.
Area of the park = (480) (320)
C1 shows correct time after 720days.
= 153600 sq.m. Choice (c)
(it gains 720 minutes i.e. 12 hours 720 days)
C2 shows correct time after 480days.
145. A certain sum of money amounts to `1,008 in 2
∵ 1.5x = 720, x = 480 days
years and to `1,164 in 31/2 years. Find the sum
The time taken by both the watches to show the
and the rate of interest.
correct time again is LCM(720, 480) i.e. 1440
(a) `900 and 12% (b) `800 and 12%
days. Choice (c)
(c) `700 and 13% (d) `800 and 13%
143. To display 500 Wax Statuettes in a Museum, it is
Solution:
required to construct a big rectangular hall,
Let the sum be ‘P’ and the rate of Interest
allowing 22.5 m3 space per statuette. The height
be r.
of the hall is to be kept at 7.5 m, while the total
2r
inner surface area of the walls must be 1,200 P(1 + 100
) = 1008 ---------- (1)
sq.m. Then the length and breadth of the hall, 3.5r
respectively, are P(1+ ) = 1164 ------ (2)
100
(a) 62 m and 22m (b) 50 m and 30 m Taking the ratio of (1) and (2)
(c) 46 m and 35 m (d) 40 m and 32 m 2r
1+ 1008
100
3.5r =
Solution: 1+ 1164
100
Let ‘l’ be the length (in m) of the rectangular 1200
hall 156 = r
100
‘b’ be the breadth (in m) of the rectangular hall. r = 13
‘h’ be the height. (in m) of the rectangular hall. r = 13%
The total space required for 500 wax statuettes = 2(13)
(22.5) (500) = 11250 m3 P (1+ ) = 1008
100
lbh = 11250 126
lb = 1500 ---------- (1) P( ) = 1008
100

Triumphant Institute of Management Education Pvt. Ltd. (T.I.M.E.) HO: 95B, 2nd Floor, Siddamsetty Complex, Secunderabad – 500 003.
Tel : 040–27898195 Fax : 040–27847334 email : info@time4education.com website : www.time4education.com AILET2018/24
1008(100) C 200 D 400
P=
126 = , =
D 20 S 150
P = `800.
Choice (d)
C 10 D 8
 = , =
146. Azhar can row 9.333 kmph in still waters and D 1 S 3
finds that it takes him thrice as much time to row C : D : S = 80 : 8 : 3.
up than as to row down the same distance in the  The triple ratio of the money spent by the three
river. The speed of the current of the river is committee’s = 80 : 8 : 3.
(a) 3. 333 km/hr (b) 3.111 km/hr Choice (a)
(c) 4.666 km/hr (d) 4.5 km/hr
149. A wooden almirah is sold at a certain price. By
Solution: selling it at 2/3 of that price one loses 10%. Find
Let the speed of Azhar in still water be ss and the the profit per cent at the original price.
speed of the current of the river be sr. (a) 15% (b) 25%
given that ss = 9.333 kmph. (c) 35% (d) 45%
Let ‘t’ be the time taken to row down a distance
of ‘d’ km. Solution:
d Let the selling price of the wooden almirah be ‘S’
then = ss + sr ------ (1) and the cost price be ‘C’.
t
d 2S
= ss – sr ------ (2) When sold at , a loss of 10% was incurred.
3t 3
Solving (1) and (2) 2s
c–
ss + sr = 3ss – 3sr 3
 × 100 = 10
4sr = 2ss c
9.333 2S 9C
sr = = 4.666 km/hr. =
2 3 10
Choice (c) 27
S= C
20
147. A wheel that has 6 cogs is meshed with a larger The profit percentage when sold at the original
S−C
wheel of 14 cogs. When the smaller wheel has price = ×(100)
made 21 revolutions, then the number of C
27
revolutions made by the larger wheel is C -C
(a) 7 (b) 9 (c) 14 (d) 42 = 20 ×100
C
= 35%.
Solution: The profit percentage when sold at original
Since the wheels are meshed like gears, price is 35%.
rotational motion is transferred from one wheel to Choice (c)
the other. At the point of contact, i.e., where the
cogs are meshed, distance travelled should be 150. The ratio between the present ages of Anu and
same for both the wheels. Balbir is 5 : 3, respectively. The ratio between
Rotational distance covered by the smaller wheel Anu's age 4 years ago and Balbir’s 4 years hence
and the larger wheel should be the same. is 1 : 1. What is the ratio between Anu's age 4
6 × 21 = 14 × x years hence and Balbir’s age 4 years ago ?
x = 9. (a) 4 : 1 (b) 2 : 4
Choice (b) (c) 3 : 2 (d) 3 : 1

148. In a University, for each `200 spent by the Solution:


Given the ratio of present ages of Anu and
Cultural Committee, Debating Committee spends
Balbir = 5 : 3.
`20 and for every `400 spent by the Debating
Let Anu’s present age be 5x.
Committee, the Student Welfare Committee
Balbir’s present age be 3x.
spends `150. The triple ratio of the money spent
by the Cultural Committee to the money spent by Age Anu’s age Balbir’s age
the Debating Committee to the money spent by Present 5x 3x
the Student Welfare Committee can be 4 years ago 5x – 4 3x – 4
expressed as 4 years hence 5x + 4 3x + 4
(a) 80 : 8 : 3 (b) 60 : 8 : 3
(c) 40 : 4 : 3 (d) 20 : 4 : 3 Also given that; 5x – 4 = 3x + 4
x = 4.
Solution: The required ratio is = (5x + 4) : (3x – 4)
Let C be the amount spent by cultural committee, = 24 : 8
D be the amount spent by Debating committee. =3:1
S be the amount spent by student welfare Choice (d)
committee.

Triumphant Institute of Management Education Pvt. Ltd. (T.I.M.E.) HO: 95B, 2nd Floor, Siddamsetty Complex, Secunderabad – 500 003.
Tel : 040–27898195 Fax : 040–27847334 email : info@time4education.com website : www.time4education.com AILET2018/25

S-ar putea să vă placă și